Difference between revisions of "2019 AMC 8 Problems/Problem 3"

(Problem)
(Added copyright)
Line 1: Line 1:
===Problem===
+
===Problem 3===
 
Which of the following is the correct order of the fractions <math>\frac{15}{11},\frac{19}{15},</math> and <math>\frac{17}{13},</math> from least to greatest?  
 
Which of the following is the correct order of the fractions <math>\frac{15}{11},\frac{19}{15},</math> and <math>\frac{17}{13},</math> from least to greatest?  
  
 
<math>\textbf{(A) }\frac{15}{11}< \frac{17}{13}< \frac{19}{15}  \qquad\textbf{(B) }\frac{15}{11}< \frac{19}{15}<\frac{17}{13}    \qquad\textbf{(C) }\frac{17}{13}<\frac{19}{15}<\frac{15}{11}    \qquad\textbf{(D) } \frac{19}{15}<\frac{15}{11}<\frac{17}{13}  \qquad\textbf{(E) }  \frac{19}{15}<\frac{17}{13}<\frac{15}{11}</math>
 
<math>\textbf{(A) }\frac{15}{11}< \frac{17}{13}< \frac{19}{15}  \qquad\textbf{(B) }\frac{15}{11}< \frac{19}{15}<\frac{17}{13}    \qquad\textbf{(C) }\frac{17}{13}<\frac{19}{15}<\frac{15}{11}    \qquad\textbf{(D) } \frac{19}{15}<\frac{15}{11}<\frac{17}{13}  \qquad\textbf{(E) }  \frac{19}{15}<\frac{17}{13}<\frac{15}{11}</math>
  
===Solution===
+
===Solution 1===
 
Consider subtracting 1 from each of the fractions. Our new fractions would then be <math>\frac{4}{11}, \frac{4}{15}, and \frac{4}{13}</math>. Since <math>\frac{4}{15}<\frac{4}{13}<\frac{4}{11}</math>, it follows that the answer is <math>\boxed{\qquad\textbf{(E) }}  \frac{19}{15}<\frac{17}{13}<\frac{15}{11}</math>
 
Consider subtracting 1 from each of the fractions. Our new fractions would then be <math>\frac{4}{11}, \frac{4}{15}, and \frac{4}{13}</math>. Since <math>\frac{4}{15}<\frac{4}{13}<\frac{4}{11}</math>, it follows that the answer is <math>\boxed{\qquad\textbf{(E) }}  \frac{19}{15}<\frac{17}{13}<\frac{15}{11}</math>
 +
 +
 +
==See Also==
 +
{{AMC8 box|year=2019|num-b=2|num-a=4}}
 +
 +
{{MAA Notice}}

Revision as of 13:27, 20 November 2019

Problem 3

Which of the following is the correct order of the fractions $\frac{15}{11},\frac{19}{15},$ and $\frac{17}{13},$ from least to greatest?

$\textbf{(A) }\frac{15}{11}< \frac{17}{13}< \frac{19}{15}  \qquad\textbf{(B) }\frac{15}{11}< \frac{19}{15}<\frac{17}{13}    \qquad\textbf{(C) }\frac{17}{13}<\frac{19}{15}<\frac{15}{11}    \qquad\textbf{(D) } \frac{19}{15}<\frac{15}{11}<\frac{17}{13}   \qquad\textbf{(E) }   \frac{19}{15}<\frac{17}{13}<\frac{15}{11}$

Solution 1

Consider subtracting 1 from each of the fractions. Our new fractions would then be $\frac{4}{11}, \frac{4}{15}, and \frac{4}{13}$. Since $\frac{4}{15}<\frac{4}{13}<\frac{4}{11}$, it follows that the answer is $\boxed{\qquad\textbf{(E) }}   \frac{19}{15}<\frac{17}{13}<\frac{15}{11}$


See Also

2019 AMC 8 (ProblemsAnswer KeyResources)
Preceded by
Problem 2
Followed by
Problem 4
1 2 3 4 5 6 7 8 9 10 11 12 13 14 15 16 17 18 19 20 21 22 23 24 25
All AJHSME/AMC 8 Problems and Solutions

The problems on this page are copyrighted by the Mathematical Association of America's American Mathematics Competitions. AMC logo.png